LSAT and Law School Admissions Forum

Get expert LSAT preparation and law school admissions advice from PowerScore Test Preparation.

 Administrator
PowerScore Staff
  • PowerScore Staff
  • Posts: 8916
  • Joined: Feb 02, 2011
|
#23573
Complete Question Explanation

Weaken—PR. The correct answer choice is (E)

The "principle" says that we can explain policy changes by looking at the economic winners only if we also show the motivation and mechanism by which the winners influenced the change.

If we couple this with a premise which shows that the "powerful foreign companies" did not have a motivation and mechanism for influencing the policy change, or perhaps that some other causal force influenced the policy change, then we can successfully defeat the "claim."

Answer choice (A): This answer choice does not undermine the idea that "foreign companies" were economic winners from the policy change. It also does not really deal with the mechanism of influence. This is not the best answer choice.

Answer choice (B): That importers have suffered is completely consistent with foreign companies benefiting. This answer choice does not defeat the claim.

Answer choice (C): The economic effects may have been impossible to predict in the short term, but perhaps the economic effects (including benefit to foreign companies) were predictable in the long term. This does not eliminate the possibility that the foreign companies had motivation to seek the policy change. This is not a good answer choice.

Answer choice (D): That the "foreign companies" compete fiercely with each other in the marketplace does not eliminate the possibility that they could band together in the political arena to lobby for tariff reductions.

Answer choice (E): This is the correct answer choice.
If we have no evidence of the mechanism by which the foreign companies induced the change, then according to the "principle", we cannot prove the claim.
 reop6780
  • Posts: 265
  • Joined: Jul 27, 2013
|
#12371
I did not have much problem finding the right answer, but cannot tell what kind of question this is.
 Lucas Moreau
PowerScore Staff
  • PowerScore Staff
  • Posts: 216
  • Joined: Dec 13, 2012
|
#12374
Hi, Reop,

This is a Weaken question, of the Principle subtype. This is an unusual question construction, I agree with you there! :0 What it's really asking can be summed up as:

Principle + Answer Choice = Argument that counters (weakens) Claim

Hope that helps,
Lucas Moreau
PowerScore

Get the most out of your LSAT Prep Plus subscription.

Analyze and track your performance with our Testing and Analytics Package.